3
$\begingroup$

I am very curious about the following condition: let $\left\{f_n \right\}_{n\in \mathbb{N}}$ be a sequence of measurable functions on a measure space $(X, \mathcal{F}, \mu)$ such that for every $\varepsilon > 0$ $$ \sum_{n \in \mathbb{N}} \mu \left( \left\{ x \in X \,\colon\, \vert f_n (x)\vert > \varepsilon \right\} \right) < \infty. $$

Why is this condition meaningful? What can we say about the functions $(f_n)$ based on it?

Note: In particular, I have been told that this gives us a variant of Egorov's theorem: for all $\delta > 0$ there is a set $E$ of measure smaller than $\delta$ on the complement of which $(f_n)$ converges uniformly to $0$. This surprises me and seems extremely strong...

$\endgroup$
2
  • $\begingroup$ By the same argument as the Borel-Cantelli theorem from probability theory, on a general measure space this condition implies for all $\epsilon>0$ that $\mu(\{x \in X: |f_n(x)|>\epsilon\quad i.o.\})=0$, in particular $\mu(\cap_{n=1}^{\infty}\cup_{k=n}^{\infty} \{x\in X: |f_k(x)|> \epsilon\})=0$. $\endgroup$
    – Michael
    Commented 20 hours ago
  • $\begingroup$ For probability spaces ($\mu(X)=1$ and each $x \in X$ is an "outcome") we get that random variables $f_n$ converge to $0$ almost surely. In probability spaces we often don't care about the concept of "uniform convergence," although the Egorov result still holds here (uniform over all outcomes in a set $E^c$). $\endgroup$
    – Michael
    Commented 18 hours ago

1 Answer 1

3
$\begingroup$

Yes. The proof is similar to the Borel-Cantelli theorem of probability theory. It can be viewed as a refinement of the standard statement of Borel-Cantelli.


Claim: Let $(X, \mathcal{F}, \mu)$ be a measure space with measure $\mu:\mathcal{F}\rightarrow [0,\infty]$. For each $n\in\{1, 2, 3, ...\}$ let $f_n:X\rightarrow\mathbb{R}$ be a measurable function. Suppose for all $\epsilon>0$ we have $$ \sum_{n=1}^{\infty} \mu(\{x \in X: |f_n(x)|> \epsilon\})<\infty $$ Then for all $\delta>0$, there is a set $E$ such that $\mu(E)\leq \delta$ and $f_n(x)$ converges uniformly to $0$ for all $x \in E^c$.

Proof: For positive integers $n,k$ define $$ q_n(k) = \sum_{i=n}^{\infty} \mu(\{x \in X: |f_i(x)|> 1/k\})$$ Comparing with our assumption, if we define $\epsilon=1/k$ then $q_n(k)$ can be viewed as the tail in the infinite sum. The assumption that the infinite sum is finite then implies that for all positive integers $k$ we have $$\lim_{n\rightarrow\infty} q_n(k)=0 \quad (*)$$

For positive integers $n, k$ define $$ A_{n,k} = \cup_{i=n}^{\infty} \{x \in X: |f_i(x)|> 1/k\}$$ Then by the union bound: $$ \mu(A_{n,k}) \leq \sum_{i=n}^{\infty} \mu(\{x \in X: |f_i(x)|>1/k\}) = q_n(k)$$ Fix $\delta>0$. By Equation (*), we know that for each $k \in \{1, 2, 3, ...\}$ we can choose a positive integer $n_k$ such that $$ q_{n_k}(k)\leq \frac{\delta}{2^k}$$ Define $$E = \cup_{k=1}^{\infty} A_{n_k,k}$$ Then by the union bound: \begin{align} \mu(E) &\leq \sum_{k=1}^{\infty} \mu(A_{n_k,k})\\ &\leq \sum_{k=1}^{\infty} q_{n_k}(k) \\ &\leq \sum_{k=1}^{\infty} \frac{\delta}{2^k}\\ &=\delta \end{align} Further, it holds that $f_n(x)$ converges uniformly to $0$ for all $x \in E^c$. This is because $$ E^c = \cap_{k=1}^{\infty}A_{n_k,k}^c = \cap_{k=1}^{\infty}\cap_{i=n_k}^{\infty}\{x\in X: |f_i(x)|\leq 1/k\}$$ This means that for any $\epsilon>0$, we can choose $k$ such that $1/k\leq \epsilon$ and we know that if $i\geq n_k$ then $|f_i(x)|\leq \epsilon$ for all $x \in E^c$. $\Box$


Regarding your question "why is this condition meaningful," you can see from the proof that it implies certain tail measures decay to zero. The condition works even when $\mu(X)=\infty$ (the statement of Egorov's theorem uses a different condition that needs $\mu(X)$ to be finite).

$\endgroup$
0

You must log in to answer this question.

Start asking to get answers

Find the answer to your question by asking.

Ask question

Explore related questions

See similar questions with these tags.